3
$\begingroup$

all. I ran into this problem and was wondering where my logic failed. I was solving an absolute value involving a quotient and went about it the following way:

$$ \begin{eqnarray} \lvert \frac{x+1}{x-2} \rvert &<& 3\\ \Rightarrow \frac{x+1}{x-2} &>& -3 \hspace{1mm}and\hspace{1mm}\frac{x+1}{x-2} < 3\\ x+1&>&-3(x-2)\hspace{2cm}\text{(Solving for the left inequality)}\\ x+1&>&-3x+6\\ 4x&>&5\\ x&>&\frac{5}{4} \end{eqnarray} $$ which cannot be true, as $x$ can equal $2$ with these restrictions which is obviously not allowed. Doing the left side the correct way:

$$ \begin{eqnarray} \frac{x+1}{x-2} &>& -3\\ \frac{x+1}{x-2} + 3 &>& 0\\ \frac{x+1}{x-2} + \frac{3x-6}{x-2} &>& 0\\ \frac{4x-5}{x-2} &>& 0\\ \Rightarrow x < \frac{5}{4}\lor x>2\\ \end{eqnarray} $$

leads to the solution set $(-\infty,\frac{5}{4})\cup(2,\infty)$. My question is, what happened with the first method where I failed to come up with a solution for $(2,\infty)$, (and why the signage for $x>\frac{5}{4}$ is backwards/incorrect in the first example, does the multiplication by -3 reverse the inequality even if I'm not introducing a new negative term to one side?). I assume since you cannot divide by zero, you're not allowed to multiply the denominator to the other side without restrictions, but I am not quite sure. Thank you, I greatly appreciate it.

$\endgroup$
3
  • 1
    $\begingroup$ The use of the Union symbol $\cup$ is certainly not the correct way, as it is used to denote union of sets. You can use a $\textrm{or}$, or the or symbol $\vee$ (\vee). $\endgroup$ Jul 7, 2021 at 23:31
  • 1
    $\begingroup$ @ultralegend5385: When writing in the context of mathematical logic, I think \lor, \land and \lnot are easier to remember for $\lor,\land,\lnot$ respectively. Then again, sometimes I prefer writing \neg over \lnot for the logical not $\neg$ $\endgroup$ Jul 7, 2021 at 23:44
  • $\begingroup$ @ultralegend5385 fixed. $\endgroup$
    – js2822
    Jul 7, 2021 at 23:54

3 Answers 3

3
$\begingroup$

In the first method, when you go from $\dfrac{x+1}{x-2}\gt -3$ to $x+1\gt -3(x-2)$, what you're doing is multiplying both sides of the inequality by $x-2$ and you assume that $x-2\gt 0$ when doing that because otherwise the inequality sign flips to $\le$

So, assuming $x\gt 2$, you get from that inequality that $x\gt 5/4$ and the second inequality $\dfrac{x+1}{x-2}\lt 3$ gives $x\gt 7/2$

So, assuming $x\gt 2$, you get $x\gt 5/4$ and $x\gt 7/2$; together they imply $x\gt\max\{2,5/4,7/2\}=7/2$

Similarly, assume $x-2\lt 0$, ie, $x\lt 2$ and solve $x+1\lt -3(x-2)$ and $x+1\gt 3(x-2)$ simultaneously to obtain the other solution set. You get $x\lt\min\{2,5/4,7/2\}=5/4$

The complete solution set is thus $(-\infty,5/4)\cup (7/2,\infty)$


The answer you arrived at is wrong. Note that $x=3\in (2,\infty)$ doesn't satisfy the original equation.

In the second method, you're only solving one of the two inequalities that are supposed to hold. If you solve the second inequality $\dfrac{x+1}{x-2}\lt 3$ in a similar way, you get $$\frac{x+1}{x-2}-3\lt 0\iff\frac{7-2x}{x-2}\lt 0\iff x\in (-\infty, 2)\cup (7/2,\infty)$$

and the solution set to your original problem is the intersection of this with $(-\infty,5/4)\cup (2,\infty)$ so that

$$[(-\infty,2)\cup (7/2,\infty)]\cap[(-\infty,5/4)\cup (2,\infty)]=(-\infty,5/4)\cup (7/2,\infty)$$

which is the solution set to the original problem $\left|\dfrac{x+1}{x-2}\right|\lt 3$

$\endgroup$
2
  • $\begingroup$ I understand now. Thank you for explaining it so well. $\endgroup$
    – js2822
    Jul 7, 2021 at 23:55
  • $\begingroup$ @js2822: You're welcome. Also, nice work for asking a good question and being responsive to others! +1 ;) $\endgroup$ Jul 7, 2021 at 23:58
2
$\begingroup$

HINT

Since both sides are nonnegative, you can square them in order to obtain an equivalent inequation: \begin{align*} \left|\frac{x+1}{x-2}\right| < 3 & \Longleftrightarrow \left(\frac{x+1}{x-2}\right)^{2} < 9\\\\ & \Longleftrightarrow \frac{(x^{2} + 2x + 1) - 9(x^{2} - 4x + 4)}{(x-2)^{2}} < 0\\\\ & \Longleftrightarrow \frac{-8x^{2} + 38x - 35}{(x-2)^{2}} < 0\\\\ & \Longleftrightarrow \begin{cases} 8x^{2} - 38x + 35 > 0\\\\ x\neq 2 \end{cases} \end{align*}

Can you take it from here?

$\endgroup$
2
  • $\begingroup$ One question: Does squaring an absolute value add any extraneous solutions in certain circumstances? Or do they never appear since the absolute value is always nonnegative? $\endgroup$
    – js2822
    Jul 8, 2021 at 0:00
  • $\begingroup$ As long as both sides are nonnegative, you can proceed as suggested so that there won't be any extraneous solutions. $\endgroup$
    – user0102
    Jul 8, 2021 at 0:46
1
$\begingroup$

We can only "flip the sign" if what is inside the absolute value is negative. And similarly, we "keep the sign", if what is inside the absolute value bracket is positive.

We could write it this way...

$-3<\frac {x+1}{x-2}<0$ or $0\le \frac {x+1}{x-2}<3$

Or say, the left-hand equation only applies when $-1<x<2$

Then we can solve the two sets of inequalities:

$-1<x<\frac 54$ or $x > \frac {7}{2}$ or $x \le -1$

Now we can combine the intervals

$(-\infty, \frac 54)\cup (\frac 72,\infty)$

$\endgroup$

You must log in to answer this question.

Not the answer you're looking for? Browse other questions tagged .